Đến nội dung

Hoang Tung 126 nội dung

Có 1000 mục bởi Hoang Tung 126 (Tìm giới hạn từ 24-05-2020)



Sắp theo                Sắp xếp  

#567714 Topic tổng hợp một số bất đẳng thức trong kì thi MO các nước

Đã gửi bởi Hoang Tung 126 on 23-06-2015 - 20:07 trong Bất đẳng thức - Cực trị

Bài 144(Poland MO): Cho a,b,c>0. CMR: nếu ab+bc+ca=1 thì

$\frac{(a^2+1)^2}{bc(b+c)}+\frac{(b^2+1)^2}{ca(c+a)}+\frac{(c^2+1)^2}{ab(a+b)}\geq 8(\sqrt{ab}+\sqrt{bc}+\sqrt{ca})$

Ta có : $a^2+1=a^2+ab+bc+ac=(a+b)(a+c)$

 

Theo Cosi và BDT  $(x+y)(y+z)(x+z)\geq \frac{8(\sum x)(\sum xy)}{9}$$(x+y)^3\geq 4xy(x+y)$ ta có :

 

  $\sum \frac{(a^2+1)^2}{bc(b+c)}=\sum \frac{\left [ (a+b)(a+c) \right ]^2}{bc(b+c)}\geq 3\sqrt[3]{\frac{(a+b)^4(b+c)^4(c+a)^4}{(abc)^2(a+b)(b+c)(c+a)}}=3\sqrt[3]{\frac{(a+b)^3(b+c)^3(c+a)^3}{(abc)^2}}\geq 3\sqrt[3]{\frac{4ab(a+b).4bc(b+c).4ac(a+c)}{(abc)^2}}=3\sqrt[3]{64(a+b)(b+c)(c+a)}\geq 12\sqrt[3]{\frac{8(\sum a)(\sum ab)}{9}}=12\sqrt[3]{\frac{8(\sum a).1}{9}}\geq 12\sqrt[3]{\frac{8\sqrt{3(\sum ab)}}{9}}=12\sqrt[3]{\frac{8\sqrt{3}}{9}}=24\sqrt[3]{\frac{1}{\sqrt{27}}}=\frac{24}{\sqrt{3}}=8\sqrt{3}= > \sum \frac{(a^2+1)^2}{bc(b+c)}\geq 8\sqrt{3}$   (1)

 

 Theo Bunhia có :$8(\sum \sqrt{ab})\leq 8\sqrt{3(\sum ab)}=8\sqrt{3.1}=8\sqrt{3}$  (2)

 

  Từ (1),(2) $= > \sum \frac{(a^2+1)^2}{bc(b+c)}\geq 8(\sum \sqrt{ab})$

 

  Dấu = xảy ra khi $a=b=c=\frac{1}{\sqrt{3}}$




#567159 $A=\frac{a+b+c+4abc}{1+4(ab+bc+ac)}$

Đã gửi bởi Hoang Tung 126 on 20-06-2015 - 20:21 trong Bất đẳng thức - Cực trị

 Bài toán: Cho các số thực dương $a,b,c> 0$ thỏa mãn $a^2+b^2+c^2=\frac{1-16abc}{4}$.

    Tìm giá trị nhỏ nhất của biểu thức:

 

         $A=\frac{a+b+c+4abc}{1+4(ab+bc+ac)}$




#567158 Topic tổng hợp một số bất đẳng thức trong kì thi MO các nước

Đã gửi bởi Hoang Tung 126 on 20-06-2015 - 20:11 trong Bất đẳng thức - Cực trị

Bài 132:(IMO 1984) Cho $a,b,c>0$ thoả mãn $a+b+c=1$.Chứng minh rằng $0\leq ab+bc+ca-2abc\leq \frac{7}{27}$

   Ta có :$\frac{1}{a}+\frac{1}{b}+\frac{1}{c}\geq \frac{9}{a+b+c}=\frac{9}{1}=9$

$= > \frac{ab+bc+ac}{abc}\geq 9abc= > ab+bc+ac-2abc\geq 7abc> 0$

$= > ab+bc+ac-2abc> 0$  (1)

 

  Đồng nhất $a+b+c=1,(a+b+c)^3=1$

 

BDT $ab+bc+ac-2abc\leq \frac{7}{27}< = > (ab+bc+ac)(a+b+c)-2abc\leq \frac{7(a+b+c)^3}{27}$

$< = > 27ab(a+b)+27bc(b+c)+27ac(a+c)+27abc\leq 7(a+b+c)^3$

$< = > 27ab(a+b)+27bc(b+c)+27ac(a+c)+27abc\leq 7(a^3+b^3+c^3)+21ab(a+b)+21bc(b+c)+21ac(a+c)+42abc$

$< = > 7(a^3+b^3+c^3)+15abc\geq 6ab(a+b)+6bc(b+c)+6ac(a+c)$

 

 Theo BDT Schur bậc 3 ta có : $6(a^3+b^3+c^3+3abc)\geq 6(ab(a+b)+bc(b+c)+ac(a+c))$

 

 Theo Cosi ta có : $a^3+b^3+c^3\geq 3abc$

 

Cộng theo vế 2 BDT và rút gọn $= > 7(a^3+b^3+c^3)+15abc\geq 6ab(a+b)+6bc(b+c)+6ac(a+c)$ (2)

 

  Từ (1),(2) $= > 0< ab+bc+ac-2abc\leq \frac{7}{27}$ .Dấu = xảy ra khi $a=b=c=\frac{1}{3}$




#566968 Topic tổng hợp một số bất đẳng thức trong kì thi MO các nước

Đã gửi bởi Hoang Tung 126 on 19-06-2015 - 21:14 trong Bất đẳng thức - Cực trị

Ở đây có được đăng các bài có trong các tạp chí không vậy, nếu như mỗi MO ko thì nghe vẻ hơi ít




#566843 Topic tổng hợp một số bất đẳng thức trong kì thi MO các nước

Đã gửi bởi Hoang Tung 126 on 19-06-2015 - 13:12 trong Bất đẳng thức - Cực trị

Bài 115 (CĐTMO 2006) : Chứng minh rằng với mọi số thực $x,y,z$ thuộc đoạn $[1;2]$ ta luôn có bất đẳng thức :
                                          $(x+y+z)(\frac{1}{x}+\frac{1}{y}+\frac{1}{z})\geq 6(\frac{x}{y+z}+\frac{y}{x+z}+\frac{z}{x+y}).$

                                           Hỏi đẳng thức xảy ra khi và chỉ khi nào ? 

 

  Không mất tổng quát giả sử $x\geq y\geq z$ .Do $2\geq x,y,z\geq 1= > \left\{\begin{matrix} x+y\geq z & & \\ y+z\geq x & & \\ z+x\geq y & & \end{matrix}\right.$

 

BĐT $< = > (\sum x)(\sum \frac{1}{x})\geq 6(\sum \frac{x}{y+z})< = > (\sum x)(\sum \frac{1}{x})-9\geq 6(\sum \frac{x}{y+z}-\frac{3}{2})$

 

 Ta có :$(\sum x)(\sum \frac{1}{x})-9=\sum \frac{(y-z)^2}{yz}$

            $\sum \frac{x}{y+z}-\frac{3}{2}=\sum \frac{(y-z)^2}{2(y+x)(z+x)}$

 

 Do đó BĐT $< = > \sum \frac{(y-z)^2}{yz}\geq 6(\sum \frac{(y-z)^2}{2(y+x)(z+x)})$

$< = > \sum \frac{(y-z)^2}{yz}\geq 3\sum \frac{(y-z)^2}{(y+x)(z+x)}< = > \sum (y-z)^2(\frac{1}{yz}-\frac{3}{(y+x)(z+x)})\geq 0$

$< = > (x-y)^2(\frac{1}{xy}-\frac{3}{(x+z)(y+z)})+(y-z)^2(\frac{1}{yz}-\frac{3}{(y+x)(z+x)})+(x-z)^2(\frac{1}{xz}-\frac{3}{(x+y)(z+y)})\geq 0$

 

  Đặt $\left\{\begin{matrix} S_{a}=\frac{1}{yz}-\frac{3}{(y+x)(z+x)} & & \\ S_{b}=\frac{1}{xz}-\frac{3}{(x+y)(z+y)} & & \\ S_{c}=\frac{1}{xy}-\frac{3}{(x+z)(y+z)} & & \end{matrix}\right.$

 

Ta có :$S_{b}=\frac{1}{xz}-\frac{3}{(x+y)(z+y)}=\frac{xy+yz+y^2-2xz}{xz(x+y)(z+y)}=\frac{x(y-z)+y(y+z)-xz}{xz(x+y)(z+y)}\geq \frac{x(y-z)+xy-xz}{xz(x+y)(z+y)}=\frac{2x(y-z)}{xz(x+y)(z+y)}=\frac{2(y-z)}{z(x+y)(z+y)}\geq 0= > S_{b}\geq 0$

  (Do $y+z\geq x= > y(y+z)\geq xy,y\geq z= > y-z\geq 0$)

          $S_{a}=\frac{1}{yz}-\frac{3}{(x+y)(x+z)}=\frac{x^2+xy+xz-2yz}{yz(x+y)(x+z)}=\frac{x^2+y(x-z)+z(x-y)}{yz(x+y)(x+z)}> 0= > S_{a}> 0$ (Do $x\geq z= > y(x-z)\geq 0,x\geq y= > z(x-y)\geq 0,x^2> 0= > x^2+y(x-z)+z(x-y)> 0$)

  Do đó $S_{a}+S_{b}> 0$

 

Ta có :$S_{b}+S_{c}=\frac{1}{xz}+\frac{1}{xy}-\frac{3}{(x+y)(z+y)}-\frac{3}{(x+z)(y+z)}$

$=\frac{y+z}{xyz}-\frac{3(x+z)+3(x+y)}{(x+y)(y+z)(x+z)}=\frac{y+z}{xyz}-\frac{6x+3y+3z}{(x+y)(y+z)(x+z)}$

  Ta chứng minh $S_{b}+S_{c}\geq 0< = > \frac{y+z}{xyz}\geq \frac{6x+3y+3z}{(x+y)(y+z)(x+z)}< = > (x+y)(x+z)(y+z)^2\geq xyz(6x+3y+3z)< = > (x^2+xz+xy+yz)(y^2+2yz+z^2)\geq 6x^2yz+3xy^2z+3xyz^2< = > x^2y^2+2x^2yz+x^2z^2+xy^2z+2xyz^2+xz^3+xy^3+2xy^2z+xyz^2+y^3z+2y^2z^2+yz^3\geq 6x^2yz+3xy^2z+3xyz^2< = > x^2y^2+x^2z^2+y^3z+yz^3+2y^2z^2+xz^3+xy^3\geq 4x^2yz$

               $< = > yz(y+z)^2+x^2(y^2+z^2)+x(y^3+z^3)\geq 4x^2yz$

  BĐT này đúng do theo Cosi ta có :

   $x^2(y^2+z^2)\geq x^2.2yz=2x^2yz$

   $x(y^3+z^3)\geq x.yz(y+z)\geq x^2yz$ ( Do $y+z\geq x$)

   $yz(y+z)^2\geq x^2yz$ (Do $y+z\geq x$

 

Cộng theo vế $= > yz(y+z)^2+x^2(y^2+z^2)+x(y^3+z^3)\geq 4x^2yz$ (Luôn đúng)

 

   Do đó $S_{b}+S_{c}\geq 0$

 

Từ đó $S_{b}\geq 0,S_{b}+S_{a}> 0,S_{b}+S_{c}\geq 0$ nên theo nguyên lý SOS thì BĐT đề bài được chứng minh. 

 

    Dấu = xảy ra khi $x=y=z$ hoặc $x=2y=2z$ và hoán vị của chúng




#566750 Topic tổng hợp một số bất đẳng thức trong kì thi MO các nước

Đã gửi bởi Hoang Tung 126 on 18-06-2015 - 22:16 trong Bất đẳng thức - Cực trị

Bài 114 (VMO 1999) : Xét các số thực dương $a,b$ sao cho phương trình : 

                                                                                     $ax^{3}-x^{2}+bx-1=0$

có ba nghiệm thực dương ( các nghiệm có thể bằng nhau).

 

Tìm giá trị nhỏ nhất của biểu thức $P=\frac{5a^{2}-3ab+2}{a^{2}(b-a)}$ .

 Gọi $x_{1},x_{2},x_{3}$ là 3 nghiệm dương của phương trình. 

   Theo định lý Viet cho phương trình bậc 3 ta có :

 

  $\left\{\begin{matrix} x_{1}x_{2}+x_{2}x_{3}+x_{3}x_{1}=\frac{b}{a} & & \\ x_{1}+x_{2}+x_{3}=\frac{1}{a} & & \\ x_{1}x_{2}x_{3}=\frac{1}{a} & & \end{matrix}\right.$

 

Ta có :$\frac{b}{a^2}=\frac{b}{a}.\frac{1}{a}=(\sum x_{1}x_{2})(\sum x_{1})$

           $\frac{\sum x_{1}x_{2}}{x_{1}x_{2}x_{3}}=\frac{\frac{b}{a}}{\frac{1}{a}}=b= > \sum \frac{1}{x_{1}}=b= > \frac{b}{a}=(\sum \frac{1}{x_{1}})(\sum x_{1})$

( Do $\sum x_{1}=\frac{1}{a}$)

Theo Cosi ta có :$\frac{1}{a}=x_{1}x_{2}x_{3}=\sum x_{1}\geq 3\sqrt[3]{x_{1}x_{2}x_{3}}= > (x_{1}x_{2}x_{3})^3\geq 27x_{1}x_{2}x_{3}= > x_{1}x_{2}x_{3}\geq \sqrt{27}=3\sqrt{3}$

 

  Từ đó 

$P=\frac{5a^2-3ab+2}{a^2(b-a)}$

$=\frac{\frac{5a^2-3ab+2}{a^3}}{\frac{a^2(b-a)}{a^3}}=\frac{\frac{5}{a}-\frac{3b}{a^2}+\frac{2}{a^3}}{\frac{b}{a}-1}$

$=\frac{5x_{1}x_{2}x_{3}-3(\sum x_{1})(\sum x_{1}x_{2})+2(\sum x_{1})^3}{(\sum x_{1})(\sum \frac{1}{x_{1}})-1}$

$=(x_{1}x_{2}x_{3})(\frac{5x_{1}x_{2}x_{3}-3(\sum x_{1})(\sum x_{1}x_{2})+2(\sum x_{1})^3}{(\sum x_{1})(\sum x_{1}x_{2})-x_{1}x_{2}x_{3}})$

$\geq 3\sqrt{3}.(\frac{5x_{1}x_{2}x_{3}-3\sum x_{1}x_{2}(x_{1}+x_{2})-9x_{1}x_{2}x_{3}+2(\sum x_{1})^3}{\sum x_{1}x_{2}(x_{1}+x_{2})+2x_{1}x_{2}x_{3}})$

$= > P\geq 3\sqrt{3}.\left [ \frac{-4x_{1}x_{2}x_{3}-3\sum x_{1}x_{2}(x_{1}+x_{2})+2(\sum x_{1})^3}{\sum x_{1}x_{2}(x_{1}+x_{2})+2x_{1}x_{2}x_{3}} \right ]$

 

Ta sẽ chứng minh:   $P\geq 12\sqrt{3}$

$< = > \frac{-4x_{1}x_{2}x_{3}-3\sum x_{1}x_{2}(x_{1}+x_{2})+2(\sum x_{1})^3}{\sum x_{1}x_{2}({x_{1}+x_{2}})+2x_{1}x_{2}x_{3}}\geq 4$

$< = > 2(\sum x_{1})^3-3\sum x_{1}x_{2}(x_{1}+x_{2})-4x_{1}x_{2}x_{3}\geq 4\sum x_{1}x_{2}(x_{1}+x_{2})+8x_{1}x_{2}x_{3}$

$< = > 2\sum x_{1}^3+6\sum x_{1}x_{2}(x_{1}+x_{2})+12x_{1}x_{2}x_{3}-3\sum x_{1}x_{2}(x_{1}+x_{2})-4x_{1}x_{2}x_{3}$

$\geq 4\sum x_{1}x_{2}(x_{1}+x_{2})+8x_{1}x_{2}x_{3}$

$< = > 2\sum x_{1}^3\geq \sum x_{1}x_{2}(x_{1}+x_{2})$

$< = > \sum (x_{1}+x_{2})(x_{1}-x_{2})^2\geq 0$

 ( Điều này luôn đúng với $x_{1},x_{2},x_{3}> 0$)

 

  Do đó $P\geq 12\sqrt{3}= > P_{min}=12\sqrt{3}< = > x_{1}=x_{2}=x_{3}< = > a=\frac{1}{3\sqrt{3}},b=\sqrt{3}$




#566521 $\frac{xz}{y^{2}+yz}+\frac{...

Đã gửi bởi Hoang Tung 126 on 17-06-2015 - 22:17 trong Bất đẳng thức và cực trị

Bài toán: Cho các số thực dương $x,y,z$ thỏa mãn $x\geq z$ . Tìm giá trị nhỏ nhất của:

 

 

 

$\frac{xz}{y^{2}+yz}+\frac{y^{2}}{xz+yz}+\frac{x+2z}{x+z}$

Đặt $\frac{x}{y}=a,\frac{y}{z}=b= > \frac{x}{z}=ab\geq 1$( Do $x\geq z= > \frac{x}{z}\geq 1$)

 

Ta có :$P=\frac{xz}{y^2+yz}+\frac{y^2}{xz+yz}+\frac{x+2z}{x+z}=\frac{1}{\frac{y}{x}.\frac{y}{z}+\frac{y}{x}}+\frac{1}{\frac{x}{y}.\frac{z}{y}+\frac{z}{y}}+\frac{1}{1+\frac{x}{z}}+1$

$=\frac{1}{\frac{b}{a}+\frac{1}{a}}+\frac{1}{\frac{a}{b}+\frac{1}{b}}+\frac{1}{1+ab}+1$

$=\frac{a}{b+1}+\frac{b}{a+1}+\frac{1}{ab+1}+1=\frac{a^2}{ab+a}+\frac{b^2}{ab+b}+\frac{1}{ab+1}+1$

$\geq \frac{(a+b)^2}{2ab+a+b}+\frac{1}{1+\frac{(a+b)^2}{4}}+1$

$\geq \frac{(a+b)^2}{2.\frac{(a+b)^2}{4}+a+b}+\frac{4}{4+(a+b)^2}+1=\frac{2(a+b)}{a+b+2}+\frac{4}{4+(a+b)^2}+1$

$= > P\geq \frac{2(a+b)}{a+b+2}+\frac{4}{4+(a+b)^2}+1$

  (Do áp dụng bđt Bunhiacopxki và bđt $mn\leq \frac{(m+n)^2}{4}$)

 

  Đặt  $t=a+b> 0= > t\geq 2\sqrt{ab}\geq 2\sqrt{1}=2= > t\geq 2$  (1)

 

 $= > P\geq \frac{2t}{t+2}+\frac{4}{t^2+4}+1$

 

 Ta chứng minh $P\geq \frac{5}{2}< = > \frac{2t}{t+2}+\frac{4}{t^2+4}+1\geq \frac{5}{2}< = > \frac{t}{t+2}+\frac{2}{t^2+4}\geq \frac{3}{4}< = > \frac{t^3+6t+4}{(t+2)(t^2+4)}\geq \frac{3}{4}< = > 4t^3+24t+16\geq 3t^3+6t^2+12t+24< = > t^3-6t^2+12t-8\geq 0$

$< = > (t-2)^3\geq 0< = > t-2\geq 0< = > t\geq 2$

  (Điều này luôn đúng do từ (1))

 

 Từ đó $P\geq \frac{5}{2}= > P_{min}=\frac{5}{2}< = > a=b=1< = > x=y=z> 0$




#566043 $P=\frac{4(a+c)}{a^2+3c^2+28}+\frac{4...

Đã gửi bởi Hoang Tung 126 on 15-06-2015 - 21:56 trong Bất đẳng thức và cực trị

Bài này cũng giống bài của bạn HoangTung:

 

Bài toán: Cho $a,b,c$ thực dương thỏa mãn $a^2+b^2+c^2=26$.Tìm GTLN của:

 

$\frac{4(a+c)}{2a^2+4bc+26}+\frac{4a}{a^2+bc+13}-\frac{4}{a(b+c)}-\frac{7}{(a+b)^2}$

 

Spoiler

Đúng vậy ,nó nằm trên mục thử sức trước kì thi ,bài này ko khó nhưng mà nó hay




#565863 $P=\frac{4(a+c)}{a^2+3c^2+28}+\frac{4...

Đã gửi bởi Hoang Tung 126 on 15-06-2015 - 10:11 trong Bất đẳng thức và cực trị

p(3,2,1)=0,428...

p(3,1,2)=0,5333....

Nói chung là bộ sô (3,1,2) hay (3,2,1) gì đó anh không nhớ




#565857 $P=\frac{4(a+c)}{a^2+3c^2+28}+\frac{4...

Đã gửi bởi Hoang Tung 126 on 15-06-2015 - 09:51 trong Bất đẳng thức và cực trị

điểm rơi (3,1,2) đúng ko a

Phải là (3,2,1) chứ nhỉ




#565842 $P=\frac{4(a+c)}{a^2+3c^2+28}+\frac{4...

Đã gửi bởi Hoang Tung 126 on 15-06-2015 - 08:26 trong Bất đẳng thức và cực trị

chuẩn đề ko a, e thấy áp dụng ghép cái 1 vs cái 3 cái 2 vs 4 mang tính tương tự

Đề siêu đúng đó em




#565841 $P=\sqrt{2\left ( a+b+c \right )}-\left (...

Đã gửi bởi Hoang Tung 126 on 15-06-2015 - 08:23 trong Bất đẳng thức và cực trị

Bài toán: Cho các số thực không âm $a,b,c$ thỏa mãn $5\left ( a^{2}+b^{2}+c^{2} \right )=6\left ( ab+bc+ca \right )$

 

Tìm giá trị lớn nhất của biểu thức: $P=\sqrt{2\left ( a+b+c \right )}-\left ( a^{2}+b^{2} \right )$

Ta có : $5(a^2+b^2+c^2)=6(ab+bc+ac)= > (a+b+c)^2+4(a^2+b^2+c^2)=8(ab+bc+ac)$  (1)

 

 Theo Cosivà Bunhia có :

 

   $8(ab+bc+ac)=8ab+8c(a+b)\leq 4(a^2+b^2)+4(c^2+(a+b)^2)=4(a^2+b^2+c^2)+4(a+b)^2\leq 4(a^2+b^2+c^2)+4.2(a^2+b^2)=4(a^2+b^2+c^2)+8(a^2+b^2)$  (2)

 

Từ (1),(2) $= > (a+b+c)^2+4(a^2+b^2+c^2)\leq 4(a^2+b^2+c^2)+8(a^2+b^2)= > (a+b+c)^2\leq 8(a^2+b^2)= > a^2+b^2\geq \frac{(a+b+c)^2}{8}$

 

Do đó $P=\sqrt{2(a+b+c)}-(a^2+b^2)\leq \sqrt{2(a+b+c)}-\frac{(a+b+c)^2}{8}$  (3)

 

   Đặt $\sqrt{2(a+b+c)}=t= > a+b+c=\frac{t^2}{2}= > (a+b+c)^2= \frac{t^4}{4}$

 

 $= > P\leq t-\frac{t^4}{32}$

 

Theo Cosi 4 số có :$\frac{t^4}{32}+\frac{1}{2}+\frac{1}{2}+\frac{1}{2}\geq 4\sqrt[4]{\frac{t^4}{32.8}}=4\sqrt[4]{\frac{t^4}{4^4}}=t= > t-\frac{t^4}{32}\leq \frac{3}{2}$

 

Từ đó $= > P\leq \frac{3}{2}= > P_{max}=\frac{3}{2}$

 

    Dấu = xảy ra khi $t=2,a=b,c=a+b= > a+b+c=2,a=b,c=a+b= > a=b=\frac{1}{2},c=1$




#565529 $P=\frac{4(a+c)}{a^2+3c^2+28}+\frac{4...

Đã gửi bởi Hoang Tung 126 on 13-06-2015 - 20:42 trong Bất đẳng thức và cực trị

   Bài toán: Cho các số thực dương $a,b,c> 0$ thay đổi thỏa mãn $a^2+b^2+c^2=14$. Tìm giá trị lớn nhất của biểu thức :

 

       $P=\frac{4(a+c)}{a^2+3c^2+28}+\frac{4a}{a^2+bc+7}-\frac{5}{(a+b)^2}-\frac{3}{a(b+c)}$

 

 

 

 

P/s: Hay 




#564854 $\frac{1}{a^3}+\frac{1}{b^3...

Đã gửi bởi Hoang Tung 126 on 10-06-2015 - 19:53 trong Bất đẳng thức - Cực trị

Bài toán: Cho các số thực dương $a,b> 0$. Tìm hằng số $k$ lớn nhất thỏa mãn bất đẳng thức sau:

 

         $\frac{1}{a^3}+\frac{1}{b^3}+\frac{k}{a^3+b^3}\geq \frac{16+4k}{(a+b)^3}$




#564828 Topic tổng hợp một số bất đẳng thức trong kì thi MO các nước

Đã gửi bởi Hoang Tung 126 on 10-06-2015 - 17:12 trong Bất đẳng thức - Cực trị

Sửa một chút về điều kiện bài toán là $a,b,c$ không âm sao cho có ít nhất một số dương và $a+b+c=ab+bc+ac$

Nếu thế thì nghe vẻ là bài toán khó hơn đó




#564748 Topic tổng hợp một số bất đẳng thức trong kì thi MO các nước

Đã gửi bởi Hoang Tung 126 on 10-06-2015 - 08:51 trong Bất đẳng thức - Cực trị

Spoiler

 

98.(Romania 2008)

 

Tìm hằng số $k$ lớn nhất để bất đẳng thức sau đúng:

 

$(a+b+c)(\frac{1}{a+b}+\frac{1}{b+c}+\frac{1}{a+c}-k)\geq k$

Trong đó $a,b,c$ thực dương thỏa $a+b+c=ab+bc+ac$

 

 

BDT $< = > k\leq \frac{(a+b+c)(\frac{1}{a+b}+\frac{1}{b+c}+\frac{1}{c+a})}{1+a+b+c}$

     $= > k\leq max(\frac{(a+b+c)(\frac{1}{a+b}+\frac{1}{b+c}+\frac{1}{c+a})}{1+a+b+c})$ (Do để tìm giá trị lớn nhất của $k$)

 

Ta chứng minh $max(\frac{(a+b+c)(\frac{1}{a+b}+\frac{1}{b+c}+\frac{1}{c+a})}{1+a+b+c})=\frac{9}{8}$

 

   Thật vậy,  BĐT 

 

$< = > 8(a+b+c)(\frac{1}{a+b}+\frac{1}{b+c}+\frac{1}{c+a})\leq 9+\frac{9(a+b+c)^2}{ab+bc+ac}$

  (Do thay $1=\frac{a+b+c}{ab+bc+ac}$)

 

 $< = > 4\left [ (a+b)+(b+c)+(c+a) \right ](\frac{1}{a+b}+\frac{1}{b+c}+\frac{1}{c+a})\leq 9+\frac{9(a+b+c)^2}{ab+bc+ac}$

$< = > 4\left [ \left [ (a+b)+(b+c)+(c+a) \right ](\frac{1}{a+b}+\frac{1}{b+c}+\frac{1}{c+a}) -9\right ]\leq9(\frac{(a+b+c)^2}{ab+bc+ac}-3)$

$< = > 8\left [ \frac{(a-b)^2}{(c+a)(c+b)}+\frac{(b-c)^2}{(b+a)(c+a)}+\frac{(c-a)^2}{(c+b)(a+b)} \right ]\leq\frac{9\left [ (a-b)^2+(b-c)^2+(c-a)^2 \right ]}{ab+bc+ac}$

$< = > (a-b)^2(\frac{9}{ab+bc+ac}-\frac{8}{(a+c)(b+c)})+(b-c)^2(\frac{9}{ab+bc+ac}-\frac{8}{(b+a)(c+a)})+(c-a)^2(\frac{9}{ab+bc+ac}-\frac{8}{(c+b)(a+b)})\geq 0$ (1)

 

 Đặt $S_{a}=\frac{9}{ab+bc+ac}-\frac{8}{(c+a)(b+a)}=\frac{ab+bc+ac+9a^2}{(\sum ab)(c+a)(b+a)}> 0$

       $S_{b}=\frac{9}{ab+bc+ac}-\frac{8}{(b+a)(b+c)}=\frac{ab+bc+ac+9b^2}{(\sum ab)(b+a)(b+c)}> 0$

      $S_{c}=\frac{9}{ab+bc+ac}-\frac{8}{(c+a)(c+b)}=\frac{ab+bc+ac+9c^2}{(\sum ab)(c+a)(c+b)}> 0$

 

Do đó $S_{a}> 0,S_{b}> 0,S_{c}> 0$ nên BĐT (1) đúng và ta có ĐPCM.

 

    Vậy $k_{max}=\frac{9}{8}$ là giá trị cần tìm




#564737 Topic tổng hợp một số bất đẳng thức trong kì thi MO các nước

Đã gửi bởi Hoang Tung 126 on 10-06-2015 - 07:15 trong Bất đẳng thức - Cực trị

Spoiler

 

97.(VMO 2006_Bảng B)

 

Tìm số thực $k$ lớn nhất sao cho với mọi $a,b,c$ thực dương thỏa $abc=1$.Ta có BĐT sau:

 

$\frac{1}{a^2}+\frac{1}{b^2}+\frac{1}{c^2}+3k\geq (k+1)(a+b+c)$

 

 

- Chọn $a=x^2,b=c=\frac{1}{x}$

 

BĐT $< = > \frac{1}{x^4}+\frac{1}{\frac{1}{x^2}}+\frac{1}{\frac{1}{x^2}}+3k\geq (k+1)(x^2+\frac{2}{x})$

$< = > \frac{1}{x^4}+2x^2+3k\geq (k+1)(x^2+\frac{2}{x})< = > \frac{1}{x^4}+2x^2-x^2-\frac{2}{x}\geq k(x^2+\frac{2}{x}-3)$

$< = > \frac{1}{x^4}+x^2-\frac{2}{x}\geq k(x^2+\frac{2}{x}-3)< = > \frac{(x-1)^2(x^2+x+1)^2}{x^4}\geq k(\frac{(x-1)^2(x+2)}{x})$

 

- Với $x\not\equiv 1= > \frac{(x^2+x+1)^2}{x^4}\geq \frac{k(x+2)}{x}= > k\leq \frac{(x^2+x+1)^2}{x^3(x+2)}$

 

    $= > k\leq \lim_{x->+\infty }\frac{(x^2+x+1)^2}{x^3(x+2)}=\lim_{x->+\infty }\frac{(1+\frac{1}{x}+\frac{1}{x^2})^2}{1+\frac{2}{x}}=1$  (Do $\lim_{x->+\infty }\frac{1}{x}=0$)

 

  Từ đó $k\leq 1= > > k_{max}=1$. Ta chứng minh $k=1$ là giá trị lớn nhất thỏa mãn bài toán

 

 Thật vậy ,thay $k=1$ vào BĐT 

 

   $= > \frac{1}{a^2}+\frac{1}{b^2}+\frac{1}{c^2}+3\geq 2(a+b+c)< = > b^2c^2+c^2a^2+a^2b^2+3\geq 2(a+b+c)$  (Do $abc=1$)

 

 Theo BĐT Schur bậc 3 ta có :$a^2b^2+b^2c^2+c^2a^2+3=(\sqrt[3]{a^2b^2})^3+(\sqrt[3]{b^2c^2})^3+(\sqrt[3]{c^2a^2})^3+3\sqrt[3]{(a^2b^2.b^2c^2.c^2a^2)}$

$\geq \sqrt[3]{a^2b^2}.\sqrt[3]{b^2c^2}(\sqrt[3]{a^2b^2}+\sqrt[3]{b^2c^2})+\sqrt[3]{b^2c^2}.\sqrt[3]{c^2a^2}(\sqrt[3]{b^2c^2}+\sqrt[3]{c^2a^2})+\sqrt[3]{c^2a^2}.\sqrt[3]{a^2b^2}(\sqrt[3]{c^2a^2}+\sqrt[3]{a^2b^2})$

$\geq \sqrt[3]{(ab^2c)^2}.2\sqrt[3]{ab^2c}+\sqrt[3]{(abc^2)^2}.2\sqrt[3]{abc^2}+\sqrt[3]{(a^2bc)^2}.2\sqrt[3]{a^2bc}$

$=\sqrt[3]{(abc)^3.b^3}+\sqrt[3]{(abc)^3.c^3}+\sqrt[3]{(abc)^3.a^3}=\sqrt[3]{a^3}+\sqrt[3]{b^3}+\sqrt[3]{c^3}=a+b+c$

 

  $= > a^2b^2+b^2c^2+c^2a^2+3\geq 2(a+b+c)$

 

 (Do áp dụng Cosi và $abc=1$)

 

  Do đó ta có ĐPCM. Dấu = xảy ra khi $a=b=c=1$




#564607 Topic tổng hợp một số bất đẳng thức trong kì thi MO các nước

Đã gửi bởi Hoang Tung 126 on 09-06-2015 - 15:37 trong Bất đẳng thức - Cực trị

Bài 95 ( USAJMO 2011 ): Cho $a,b,c$ là các số thực dương thỏa mãn : $a^{2}+b^{2}+c^{2}+(a+b+c)^{2}\leq 4$ . Chứng minh rằng : 

$\frac{ab+1}{(a+b)^{2}}+\frac{bc+1}{(b+c)^{2}}+\frac{ca+1}{(c+a)^{2}}\geq 3$

Ta có : $4\geq \sum a^2+(\sum a)^2=2(\sum a^2+\sum ab)= > 2\geq \sum a^2+\sum ab$

 

Từ đó :$\sum \frac{ab+1}{(a+b)^2}=\sum \frac{2ab+2}{2(a+b)^2}\geq \sum \frac{2ab+\sum a^2+\sum ab}{2(a+b)^2}=\sum \frac{(a+b)^2+(c+a)(c+b)}{2(a+b)^2}=\sum (\frac{1}{2}+\frac{(a+c)(c+b)}{2(a+b)^2})=\frac{3}{2}+\frac{1}{2}(\sum \frac{(c+a)(c+b)}{(a+b)^2})\geq \frac{3}{2}+\frac{1}{2}.3\sqrt[3]{\frac{(c+a)(c+b)}{(a+b)^2}.\frac{(b+c)(b+a)}{(c+a)^2}.\frac{(a+b)(a+c)}{(b+c)^2}}=\frac{3}{2}+\frac{3}{2}=3= > \sum \frac{ab+1}{(a+b)^2}\geq 3$

 

  Dấu = xảy ra khi $a=b=c=\frac{1}{\sqrt{3}}$ 




#564569 Topic tổng hợp một số bất đẳng thức trong kì thi MO các nước

Đã gửi bởi Hoang Tung 126 on 09-06-2015 - 11:22 trong Bất đẳng thức - Cực trị

Bài 93 ( IMO 2012 ) : Cho $a_{2},a_{3},...,a_{n}$ thỏa mãn : $a_{2}.a_{3}...a_{n}=1$ . Chứng minh rằng : 

$(1+a_{2})^{2}(1+a_{3})^{3}...(1+a_{n})^{n}>n^{n}$

Theo Cosi ta có : 

 

  $1+a_{2}\geq 2\sqrt{a_{2}}= > (1+a_{2})^2\geq 2^2.a_{2}$

 $a_{3}+1=a_{3}+\frac{1}{2}+\frac{1}{2}\geq 3\sqrt[3]{\frac{a_{3}}{2^2}}= > (1+a_{3})^3\geq \frac{3^3.a_{3}}{2^2}$

  .....................................................................................................

 $(a_{n}+1)=a_{n}+\frac{1}{n-1}+\frac{1}{n-1}+...+\frac{1}{n-1}\geq n\sqrt[n]{\frac{a_{n}}{(n-1)^{n-1}}}= > (1+a_{n})^n\geq n^{n}.\frac{a_{n}}{(n-1)^{(n-1)}}$

 

 Nhân theo vế các bất đẳng thức 

 

 $= > (1+a_{2})^2(1+a_{3})^3...(1+a_{n})^n\geq 2^{2}.3^{3}...n^{n}.(a_{1}a_{2}...a_{n}).(\frac{1}{2^{2}}.\frac{1}{3^{3}}...\frac{1}{(n-1)^{n-1}})=n^{n}= > (1+a_{2})^2(1+a_{3})^3...(1+a_{n})^{n}\geq n^{n}$

      (Do $a_{2}a_{3}...a_{n}=1$)

 

 Dấu = xảy ra khi $a_{2}=1,a_{3}=\frac{1}{2},...a_{n}=\frac{1}{n-1}= > a_{2}a_{3}...a_{n}=\frac{1}{2.3...(n-1)}$

 

Nhưng điều này vô lý do $a_{2}a_{3}...a_{n}=1$

 

  Do đó dấu = ko xảy ra ,tức là $(a_{2}+1)^2(a_{3}+1)^3...(a_{n}+1)^{n}> n^{n}$




#564464 Topic tổng hợp một số bất đẳng thức trong kì thi MO các nước

Đã gửi bởi Hoang Tung 126 on 08-06-2015 - 19:43 trong Bất đẳng thức - Cực trị

Bài 89: (Turkish MO 2007)

Cho $a,b,c$ thực dương thỏa mãn: $a+b+c=3$.Chứng minh bất đẳng thức:

 

$\frac{a^2+3b^2}{ab^2(4-ab)}+\frac{b^2+3c^2}{bc^2(4-bc)}+\frac{c^2+3a^2}{ca^2(4-ca)}\geq 4$

Theo Cosi ta có :

 

 $\sum \frac{a^2+3b^2}{ab^2(4-ab)}=\sum \frac{(a^2+b^2)+2b^2}{ab^2(4-ab)}\geq \sum \frac{2ab+2b^2}{ab^2(4-ab)}=\sum \frac{2b(a+b)}{ab^2(4-ab)}$

$=2\sum \frac{a+b}{ab(4-ab)}\geq 2\sum \frac{2\sqrt{ab}}{ab(4-ab)}=4\sum \frac{1}{\sqrt{ab}(4-ab)}$   (1)

 

Mà $8+ab=(4-ab)+(4-ab)+3ab\geq 3\sqrt[3]{3ab(4-ab)^2}= > (8+ab)^3\geq 81ab(4-ab)^2$

$= > ab(4-ab)^2\leq \frac{(8+ab)^3}{81}= > \sqrt{ab}(4-ab)\leq \frac{\sqrt{(8+ab)^3}}9{}= > \frac{1}{\sqrt{ab}(4-ab)}\geq \frac{9}{\sqrt{(8+ab)^3}}$

$= > \sum \frac{1}{\sqrt{ab}(4-ab)}\geq 9\sum \frac{1}{\sqrt{(8+ab)^3}}\geq 9.3\sqrt[3]{\frac{1}{\sqrt{(8+ab)^3(8+bc)^3(8+ac)^3}}}=\frac{27}{\sqrt{(8+ab)(8+bc)(8+ac)}}\geq \frac{27}{\sqrt{\frac{(8+ab+8+bc+8+ac)^3}{27}}}\geq \frac{27}{\sqrt{\frac{(24+\frac{(a+b+c)^2}{3})}{27}}}=\frac{27}{\sqrt{\frac{(25+3)^3}{27}}}=\frac{27}{\sqrt{27^2}}=1$

   $= > \sum \frac{1}{\sqrt{ab}(4-ab)}\geq 1$    (2)

 

   Từ (1) ,(2) $= > \sum \frac{a^2+3b^2}{ab^2(4-ab)}\geq 4$ và ta có ĐPCM

 

Dấu = xảy ra khi $a=b=c=1$




#563986 $P=\frac{5x+5}{x^2+5y+11}+\frac{4y+4}{y^2+4x+7}+\frac{x^2...

Đã gửi bởi Hoang Tung 126 on 06-06-2015 - 19:27 trong Bất đẳng thức và cực trị

Cho các số thực x,y thỏa mãn $2\leq x\leq 3;1\leq y\leq 3$ .Tìm GTNN của $P=\frac{5x+5}{x^2+5y+11}+\frac{4y+4}{y^2+4x+7}+\frac{x^2+3y^2}{150}$

Ta có $2\leq x\leq 3= > (x-2)(x-3)\leq 0= > x^2+6\leq 5x= > x^2+5y+11\leq 5(x+y+1)= > \frac{5x+5}{x^2+5y+11}\geq \frac{5(x+1)}{5(x+y+1)}=\frac{x+1}{x+y+1}$

 

Tương tự $1\leq y\leq 3= > (y-1)(y-3)\leq 0= > y^2+4x+7\leq 4y+4x+4=4(x+y+1)= > \frac{4y+4}{y^2+4y+7}\geq \frac{4(y+1)}{4(x+y+1)}=\frac{y+1}{x+y+1}$

 

Từ đó $= > P\geq \frac{x+1}{x+y+1}+\frac{y+1}{x+y+1}+\frac{x^2+3y^2}{150}=\frac{x+y+2}{x+y+1}+\frac{x^2+3y^2}{150}=1+\frac{1}{x+y+1}+\frac{x^2+3y^2}{150}$

 

Mà $(x+y)^2=(x+y\sqrt{3}.\frac{1}{\sqrt{3}})^2\leq (x^2+3y^2)(1+\frac{1}{3})=\frac{4(x^2+3y^2)}{3}= > x^2+3y^2\geq \frac{3(x+y)^2}{4}$

 

Do đó $P\geq 1+\frac{1}{x+y+1}+\frac{\frac{3(x+y)^2}{4}}{150}=1+\frac{1}{x+y+1}+\frac{(x+y)^2}{200}$

 

Đặt $x+y=t= > P\geq 1+\frac{1}{t+1}+\frac{t^2}{200}$

 

  Đến đây xét đạo hảm biến t ,cụ thể là sẽ chứng minh

 

 $\frac{1}{t+1}+\frac{t^2}{200}\geq \frac{7}{25}< = > (t+9)(t-4)^2\geq 0$ (Luôn đúng)

 

  Do đó $P\geq 1+\frac{7}{25}=\frac{32}{25}= > P\geq \frac{32}{25}= > P$ Min $=\frac{32}{25}$

 

 $< = > (x-2)(x-3)=(y-1)(y-3)=0,x=3y,x+y=4= > x=3,y=1$




#563910 $\sum \frac{a+b}{1-ab}\leq 3(\su...

Đã gửi bởi Hoang Tung 126 on 06-06-2015 - 11:22 trong Bất đẳng thức - Cực trị

Spoiler

 

Chứng minh BĐT tương đương sau:

 

$\frac{x+y}{3-xy}+\frac{y+z}{3-yz}+\frac{x+z}{3-xz}\leq x+y+z$ với $x^2+y^2+z^2=3$

 

Đưa về dạng đồng bậc là:

 

$\frac{x+y}{x^2+y^2+z^2-xy}+\frac{y+z}{x^2+y^2+z^2-yz}+\frac{x+z}{x^2+y^2+z^2-xz}\leq \frac{3(x+y+z)}{x^2+y^2+z^2}$

 

Bỏ qua điều kiện $x^2+y^2+z^2=3$ ta chuẩn hóa $x+y+z=1$ và đặt $p=x+y+z=1,q=xy+yz+xz,r=xyz$ :unsure:

 

Viết lại BĐT trên dưới dạng

 

$f(r)=3r^2+r(20q-16q^2-6)+16q^3-20q^2+8q-1\leq 0$

 

Hàm này nghịch biến theo $r$ nên sử dụng phép đặt $3q=1-x^2$ với $0\leq x\leq 1$ thì ta có: $r\geq \frac{(1+x)^2(1-2x)}{27}$

 

Do đó mà $f(r)\leq f(\frac{(1+x)^2(1-2x)}{27})=\frac{1}{243}(1-2x)^2(x-2)x^2(8x^2+x+2)\leq 0$ với mọi $0\leq x\leq 1$

 

Vậy BĐT được chứng minh.Lấy $x=\sqrt{3}a,y=\sqrt{3}b,z=\sqrt{3}c$ ta được BĐT ban đầu.

  Đây là cách của mình ,lời giải ko cần phức tạp như trên đâu

 

Ta có :$\frac{a+b}{1-ab}+\frac{b+c}{1-bc}+\frac{c+a}{1-ac}\leq 3(a+b+c)$

        $< = > (\frac{a+b}{1-ab}-(a+b))+(\frac{b+c}{1-bc}-(b+c))+(\frac{c+a}{1-ac}-(a+c))\leq a+b+c$

        $< = > \frac{ab(a+b)}{1-ab}+\frac{bc(b+c)}{1-bc}+\frac{ac(a+c)}{1-ac}\leq a+b+c$

 

 Theo Bunhiacopxki có :$\frac{ab(a+b)}{1-ab}=\frac{ab(a+b)}{a^2+b^2+c^2-ab}=\frac{ab(a+b)^2}{(a+b)(a^2+b^2+c^2-ab)}=\frac{ab(a+b)^2}{a^3+b^3+ac^2+bc^2}=\frac{ab(a+b)^2}{a(a^2+c^2)+b(b^2+c^2)}\leq ab.(\frac{a^2}{a(a^2+c^2)}+\frac{b^2}{b(b^2+c^2)})=\frac{a^2b}{a^2+c^2}+\frac{ab^2}{b^2+c^2}= > \frac{ab(a+b)}{1-ab}\leq \frac{a^2b}{a^2+c^2}+\frac{ab^2}{b^2+c^2}$

 

   Tương tự $\frac{bc(b+c)}{1-bc}\leq \frac{bc^2}{a^2+c^2}+\frac{b^2c}{a^2+b^2}$

                    $\frac{ac(a+c)}{1-ac}\leq \frac{a^2c}{a^2+b^2}+\frac{ac^2}{b^2+c^2}$

    

  Cộng theo  vế các BĐT 

 

$= > \frac{ab(a+b)}{1-ab}+\frac{bc(b+c)}{1-bc}+\frac{ac(a+c)}{1-ac}\leq \frac{b(a^2+c^2)}{a^2+c^2}+\frac{a(b^2+c^2)}{b^2+c^2}+\frac{c(a^2+b^2)}{a^2+b^2}=a+b+c= > \frac{ab(a+b)}{1-ab}+\frac{bc(b+c)}{1-bc}+\frac{ac(a+c)}{1-ac}\leq a+b+c$

 

   Do đó ta có ĐPCM .Dấu = xảy ra khi $a=b=c=\frac{1}{\sqrt{3}}$




#563875 $\sum \frac{a+b}{1-ab}\leq 3(\su...

Đã gửi bởi Hoang Tung 126 on 06-06-2015 - 08:51 trong Bất đẳng thức - Cực trị

Spoiler

 

Chứng minh BĐT tương đương sau:

 

$\frac{x+y}{3-xy}+\frac{y+z}{3-yz}+\frac{x+z}{3-xz}\leq x+y+z$ với $x^2+y^2+z^2=3$

 

Đưa về dạng đồng bậc là:

 

$\frac{x+y}{x^2+y^2+z^2-xy}+\frac{y+z}{x^2+y^2+z^2-yz}+\frac{x+z}{x^2+y^2+z^2-xz}\leq \frac{3(x+y+z)}{x^2+y^2+z^2}$

 

Bỏ qua điều kiện $x^2+y^2+z^2=3$ ta chuẩn hóa $x+y+z=1$ và đặt $p=x+y+z=1,q=xy+yz+xz,r=xyz$ :unsure:

 

Viết lại BĐT trên dưới dạng

 

$f(r)=3r^2+r(20q-16q^2-6)+16q^3-20q^2+8q-1\leq 0$

 

Hàm này nghịch biến theo $r$ nên sử dụng phép đặt $3q=1-x^2$ với $0\leq x\leq 1$ thì ta có: $r\geq \frac{(1+x)^2(1-2x)}{27}$

 

Do đó mà $f(r)\leq f(\frac{(1+x)^2(1-2x)}{27})=\frac{1}{243}(1-2x)^2(x-2)x^2(8x^2+x+2)\leq 0$ với mọi $0\leq x\leq 1$

 

Vậy BĐT được chứng minh.Lấy $x=\sqrt{3}a,y=\sqrt{3}b,z=\sqrt{3}c$ ta được BĐT ban đầu.

Lại chơi p,q,r hả bạn, cách này ko hay




#563804 $\sum \frac{a+b}{1-ab}\leq 3(\su...

Đã gửi bởi Hoang Tung 126 on 05-06-2015 - 21:57 trong Bất đẳng thức - Cực trị

Nếu dùng định lý ABC ở đây thì ta chỉ cần chứng minh bài toán trong 2 trường hợp là $a=b$ hay $c=0$.Bởi hàm thu được theo $abc$ là hàm nghịch biến, nhưng trường hợp $a=b$ là khá vất nếu không có máy tính phép tính có căn thức khá cồng kềnh (có lẽ có 1 lời giải nào đó đẹp hơn bằng C-S).Find it! :icon6:

Bài này ko cần dùng đến cái đó ,mà dùng đền thì rất dài và phức tạp, nói chung là dùng BĐT cơ bản là xong




#563688 $\sum \frac{a+b}{1-ab}\leq 3(\su...

Đã gửi bởi Hoang Tung 126 on 05-06-2015 - 15:47 trong Bất đẳng thức - Cực trị

   Bài toán :  Cho các số thực dương $a,b,c> 0$  thỏa mãn $a^2+b^2+c^2=1$ .CMR:

 

          $\frac{a+b}{1-ab}+\frac{b+c}{1-bc}+\frac{c+a}{1-ac}\leq 3(a+b+c)$